3
$\begingroup$

$\pi$ and $e$ are constants which we stumble on everytime we are doing math. I wonder if there is some book which exploits this fact and contains exercises whose solution involved these constants.

$\endgroup$
12

2 Answers 2

3
$\begingroup$

As requested in the comments, here is a question I came up with, and after investigating it, realised the answer has to do with one of the mathematical constants $\pi$ or $e$ (although I will not reveal which one).

Is there $a_1\in \left( \frac{1}{2}, 1 \right),$ and a sequence $a_{n+1}=(n+1)a_n-1\ \forall\ n\in\mathbb{N},\ $ such that $\frac{1}{n+1} < a_n < \frac{1}{n}\ \forall\ n\in\mathbb{N}?$

When I first came up with this problem, I thought the answer was "no", and that the problem was related to the irregularity of distributions problem. However, this is not the case, and I will leave it as an exercise to the reader to discover the truth.

$\endgroup$
6
  • $\begingroup$ I have programmed your sequence with the following Matlab code : a=exp(1)-2 % your initialization) ;A=[a]; for n=1:20 a=(n+1)*a-1 A=[A,a]; end 1./A and what I obtain is indeed a sequence whose integer parts are 1,2,3,...15, and suddenly, this sequence behaves in a foolish way. How do you explain it ? $\endgroup$
    – Jean Marie
    Commented Jul 31 at 16:52
  • $\begingroup$ @JeanMarie "I obtain a sequence with integer part $1,2,3,\ldots, 15.$" I'm not sure what you mean by this. If $a_1 = e-2,$ then $a_2=0.436\ldots,\ a_3=0.309\ldots, \ a_4=0.238\ldots.$ So they all satisfy the inequality $\frac{1}{n+1} < a_n < \frac{1}{n}\ $. To prove this will always happen, consider: $a_1=e-2 = \displaystyle\sum_{n=2}^{\infty}\frac{1}{n!}.$ $\endgroup$ Commented Jul 31 at 19:36
  • $\begingroup$ Explanation : instead of $\frac{1}{n+1} < a_n < \frac{1}{n}$, I am working on the inverses $n < \frac{1}{a_n} < n+1$; it is why I consider the integer part of $ \frac{1}{a_n} $ $\endgroup$
    – Jean Marie
    Commented Jul 31 at 20:10
  • 1
    $\begingroup$ @JeanMarie you don't need to do that though. You're doing it as an aside? $\endgroup$ Commented Aug 1 at 10:26
  • $\begingroup$ In fact, the justification is based on the following "Horner scheme" : $e-2=\frac12(1+\frac13(1+\frac14(1+(\cdots))))$ $\endgroup$
    – Jean Marie
    Commented Aug 2 at 15:33
1
$\begingroup$

Precalculus would be a good place to start

https://openstax.org/details/books/precalculus-2e/

Chapters 4 and 5.

$\endgroup$

You must log in to answer this question.

Not the answer you're looking for? Browse other questions tagged .